Unitäre irreduzible Darstellungen der kleinen Gruppe SO(3)SO(3)SO(3)

Ich arbeite mich derzeit durch die Methode der induzierten Darstellungen, um die unitären irreduziblen Darstellungen der Poincare-Gruppe zu berechnen.

Konventionen/Notation

Metrische Signatur = ( , + , + , + )

C = = 1

Die Indizes i,j,k laufen über 1,2,3, während alle anderen lateinischen Indizes über 0,1,2,3 laufen.

X A = ( T , X )

Kontext

In diesem Thread werde ich den (positiv definiten) Nicht-Null-Massenfall betrachten. Ich nehme den Standardschwung zu sein k A = ( M , 0 , 0 , 0 ) wobei m die Quadratwurzel des Eigenwerts des Casimir-Operators ist C 1 = P A P A . Die entsprechende kleine Gruppe ist H k = S Ö ( 3 ) .

Der zweite Casimir-Operator ist das Quadrat des Pauli-Lubanski-Vektors; C 2 = W A W A . Für den Standardimpuls gilt W A = ( 0 , M J ich ) Wo J ich ist der i-te Rotationsgenerator (dh um die i-te Raumachse). Daher, C 2 = M 2 J J . Aus QM kennen wir die Eigenwerte von J 2 Sind S ( S + 1 ) . Nach Schurs Lemma gilt also bei jedem Irrep der Poincare-Gruppe: C 2 = M 2 S ( S + 1 ) Ausweis . Id ist der Identitätsoperator ... \mathbb{1} funktioniert nicht.

Wenn ich also einen Wert für m festlege (und damit die Massenschalenoberfläche festlege), dann werden die unitären Irreps der Poincare-Gruppe (für dieses m) durch den 'Spin' s klassifiziert.

In diesem Fall kann die Methode der induzierten Wiederholungen wie folgt ausgedrückt werden:

U ( Λ ) | P , σ = Σ σ ' D ( S ) ( H ( Λ , P ) ) σ ' σ | Λ P , σ '
Wo H ( Λ , P ) = L 1 ( Λ P ) Λ L ( P ) H k Und L ( P ) ist die Standard-Lorentz-Transformation, die den Standardimpuls zu p bringt, dh L ( P )     B A k B = P A . Auch, D ( S ) ( H ( Λ , σ ) ) ist das unitäre Irrep des kleinen Gruppenelements H ( Λ , σ ) in der Spin-s-Darstellung.

Also wie die homogene Lorentz-Transformation zu erarbeiten U ( Λ ) wirkt auf den Staat | P , σ , müssen wir die einheitlichen Irreps der kleinen Gruppe ausarbeiten.

Per Konvention nehme ich die Standard-Lorentz-Transformation, L ( P ) , sein

L ( P )       B A = ( E P / M P J / M P ich / M δ ich J + P ich P J M ( E P + M ) ) ,                       ich , J = 1 , 2 , 3.

Frage/Lösungsversuch

Leider haben wir in meinem nicht-relativistischen QM-Kurs während des Studiums nie über die Spindarstellungen der Rotationsgruppe gesprochen S Ö ( 3 ) , also versuche ich jetzt, diese Lücke zu schließen. Ich bin ein wenig verwirrt darüber, wie wir den einheitlichen Irreps konstruieren S Ö ( 3 ) , insbesondere bin ich mir nicht sicher, wie ich die Matrixelemente berechnen soll D ( S ) ( H ( Λ , σ ) ) σ ' σ . Dies ist meine Frage, und das Folgende ist, was ich bisher habe.

Bei dem Versuch, das, was ich im Grundstudium QM gelernt habe, mit dem oben Gelernten zu übersetzen, bin ich zu folgenden Schlussfolgerungen gekommen:

  • Für den massiven Fall (und für den Standardimpuls, damit wir im Ruherahmen sind), die σ (die andere Freiheitsgrade als den 4-Impuls darstellen sollen) entsprechen der "magnetischen Quantenzahl", M J .
  • Für einen gegebenen Spin s, σ = M J = { S , S + 1 , . . , S 1 , S } . Hier nehme ich J = l + S = S da wir uns im Ruhesystem befinden und kein 'Bahndrehimpuls' ( l = 0 ). Daher ist für eine gegebene Masse m die Dimension des Irrep der kleinen Gruppe 2 S + 1 .

Definiere den 3-Vektor S ich = 1 M W ich = 1 2 ε ich J k J J k S S = S ( S + 1 ) Ausweis was der folgenden Algebra gehorcht:

[ S ich , S J ] = ich ε ich J k S k
Daher [ S ich , S 2 ] = [ 1 2 ε ich J k J J k , 1 M 2 C 2 ] = 0 (seit C 2 ist ein Casimir-Operator von ISO ( 3 , 1 ) ). Daher können wir gleichzeitig diagonalisieren S 2 und einer von S ich , sagen, S 3 .

Auf die übliche Weise können wir das ausrechnen:

S 2 | S , σ = S ( S + 1 ) | S , σ                                                               ( 1 )
S 3 | S , σ = σ | S , σ                                 ( Wo  σ = M S )       ( 2 )
Und definieren Sie Hebe-/Senkoperatoren:
S ± = S 1 ± ich S 2
S ± | S , σ = S ( S + 1 ) σ ( σ ± 1 )   | S , σ ± 1         ( 3 )

Nach Weinberg gilt für eine infinitesimale Drehung R ich k = δ ich k + Θ ich k , wir haben das

D ( S ) ( 1 + Θ ) σ ' σ = δ σ ' σ + ich 2 Θ ich k ( J ( S )   ich k ) σ ' σ

Um eine Darstellung einer endlichen Drehung zu erhalten, müssen wir den infinitesimalen Fall potenzieren:

D ( S ) ( R ) = exp { ich Θ ich k J ich k }
Was, glaube ich, in meiner Notation bedeutet:
D ( S ) ( R ( θ ) ) = exp { ich θ S }
Für eine Drehung um einen Winkel | θ | um θ , zusammen mit der Beobachtung, dass S ich = 1 2 ε ich J k J J k S = ( J 23 , J 31 , J 12 ) = ( J 1 , J 2 , J 3 ) = J .

Wir können also die unitären Darstellungen der kleinen Gruppe berechnen S Ö ( 3 ) von:

D ( S ) ( R ( θ ) ) σ ' σ = S , σ ' | e ich θ S | S , σ                         ( 4 )

Wir können dies explizit berechnen, indem wir die Gleichungen (2) und (3) verwenden, und daher ist die Größe der Matrix D für ein festes s (2s+1)(2s+1).

Ich denke, was ich bisher gesagt habe, ist größtenteils richtig (aber ich hätte gerne eine Bestätigung). Wenn das, was ich gesagt habe, tatsächlich richtig ist, dann habe ich das Problem, dass ich nicht weiß, wie man dazwischen übersetzt D ( S ) ( H ( Λ , P ) ) σ ' σ Und D ( S ) ( R ( θ ) ) σ ' σ . Zum Beispiel, gegeben (4), wie kann ich die entsprechenden Matrixelemente von berechnen D ( S ) ( H ( Λ , P ) ) σ ' σ ? Ich weiß, dass es eine Beziehung geben muss, weil sie beide nur Drehungen sind. Daher kann ich darauf schließen θ = θ ( Λ , P ) . Aber es scheint, dass die Standardmethode zur Berechnung der gewünschten Matrixelemente darin besteht, die Standard-Lorentz-Transformationen zu verwenden, die ich im Kontextabschnitt bereitgestellt habe ... aber für mich scheint dies nur nützlich zu sein, wenn wir eine explizite Methode zur Berechnung der Elemente haben der Vertretung von H ( Λ , P ) , wie Gleichung (4). Gibt es so einen expliziten Ausdruck? Beifall.

1. Ich denke, was Sie geschrieben haben, ist richtig. 2. Ich bin mir nicht sicher, was die eigentliche Frage ist. Ich denke, der beste Weg, diese Matrixelemente tatsächlich zu berechnen, wäre zu berechnen θ ( Λ , P ) . Dies scheint mühsam, aber machbar - Sie können explizit kalkulieren L 1 ( Λ P ) Λ L ( P ) , Rotationsmatrix holen und "ablesen" θ , Rechts?

Antworten (2)

Diese Antwort basiert auf diesem Artikel von A. Ungar.

Ungar hat die Thomas-Rotationsformel berechnet, die fast das ist, was Sie brauchen. Ich werde das allgemeine Verfahren beschreiben, und in einigen Fällen werde ich Sie für den Beweis an Ungar verweisen. Ich werde (genau wie Ungar) die Boosts eher in Geschwindigkeiten als in Impulsen ausdrücken. Wenn Sie möchten, können Sie die Übung mit der Impulsparametrisierung wiederholen. Von Wikipedia haben wir

B ( v ) = [ γ v γ v C v T γ v C v 1 ( 3 × 3 ) + γ v 2 1 + γ v v v T C 2 ]

Der entscheidende Punkt beim Auffinden der Wigner-Rotation ist die Beobachtung, dass jede Lorentz-Transformation (nicht eindeutig) als Produkt eines Boosts und einer Rotation zerlegt werden kann:

Λ = B ( u ) R
Jede Wahl der Zerlegungsmethode ist ausreichend, aber wir müssen mit einer festen Zerlegungsmethode arbeiten. Ich beschreibe Ihnen am Ende der Antwort eine mögliche Methode. Wenn wir nun zwei Boosts multiplizieren, erhalten wir einen Boost mit der relativistischen Additionsgeschwindigkeit + einer Rotation (oft als Thomas-Rotation bezeichnet):
B ( u ) B ( v ) = B ( u v ) T Ö M ( u , v )
Wo T Ö M ist eine Rotationsmatrix Ungar fand die allgemeine Lösung für die Thomas-Rotation (Gleichung: (13) im Artikel)
T Ö M ( u , v ) = B ( u v ) B ( u ) B ( v )
Jetzt sind wir in der Lage, die Gleichung für die Wigner-Rotation zu lösen. Wir müssen lösen:
Λ B ( v ) = B ( Λ v ) W
für W . Wir parametrisieren Λ , erhalten wir für die linke Seite:

Λ B ( v ) = B ( u ) R B ( v ) = B ( u ) R B ( v ) R 1 R = B ( u ) B ( R v ) R = B ( u R v ) T Ö M ( u , R v ) R

und für die rechte Seite

B ( Λ v ) W = B ( B ( u ) R v ) W = B ( u R v ) W

Daher:

W = T Ö M ( u , R v ) R
Was bleibt, ist eine spezifische Parametrisierung einer allgemeinen Lorentz-Matrix in einen Boost und eine Rotation zu beschreiben: We need to find B ( u ) Und R so dass:
[ λ 0 ξ T η Λ 1 ] = [ γ v γ v C v T γ v C v 1 ( 3 × 3 ) + γ v 2 1 + γ v v v T C 2 ] [ 1 0 0 R 1 ]

Das beobachten wir R 1 muss die folgende Beziehung erfüllen

ξ = R 1 η
Daher R 1 muss den 3-Vektor drehen η in den 3-Vektor ξ Die Lösung (mit einer ähnlichen Methode wie Ungar) kann wie folgt geschrieben werden:
R 1 = 1 ( 3 × 3 ) + Sünde ( θ ) Ω + ( cos ( θ 1 ) ) Ω 2

Wo θ ist der Winkel zwischen den Vektoren η Und ξ

Sünde ( θ ) = η × ξ | η | | ξ |
Und
Ω ich J = η ich ξ J ξ ich η J | η | | ξ |

Danke für die Antwort. Der Link zu dem Artikel, auf den Sie verweisen, funktioniert nicht. Könnten Sie bitte den Link reparieren oder den Namen des Artikels und die Zeitschrift notieren, in der er sich befindet? Denken Sie auch, dass diese Methode angewendet werden könnte, um die zweite Frage zu beantworten, die ich in dem unten angegebenen Beitrag habe? physical.stackexchange.com/questions/349836/…
Entschuldigung, ich habe es zweimal versucht und jedes Mal, wenn der Link zunächst funktioniert, funktioniert er dann nicht mehr. Jedenfalls ist dies der Link von einer vorherigen Seite in Google Scholar, den ich auch in den Text einfügen werde. Scholar.google.co.il/…

Um die obige Antwort zu ergänzen, haben die Spinmatrizen für beliebigen Spin kompakte explizite Ausdrücke [1][2].

( S X ) A B = 2 ( δ A , B + 1 + δ A + 1 , B ) ( S + 1 ) ( A + B 1 ) A B ( S j ) A B = ich 2 ( δ A , B + 1 δ A + 1 , B ) ( S + 1 ) ( A + B 1 ) A B 1 A , B 2 S + 1 ( S z ) A B = ( S + 1 A ) δ A , B = ( S + 1 B ) δ A , B .

Die Quellen enthalten explizite Ausdrücke für Spin 1 , 3 2 , 2 , Und 5 2 . Mit diesen können Sie potenzieren, um die explizite Darstellung jeder Rotation auf jedem Spin zu erhalten. J Objekt:

D σ ' σ ( J ) ( R ( θ ) ) = D σ ' σ ( J ) ( e ich θ S )

Im Allgemeinen ist das Ergebnis für beliebige Spin- J ist nicht so hübsch, wie es beim Spin- 1 / 2 (und wohl Spin- 1 ).

Zusammenfassend kann man mit den Ergebnissen von David Bar Mosche (oben) den Thomas-Rotationsvektor berechnen θ entsprechend der Wigner-Rotation. Dann können Sie mit diesen Ergebnissen die Wirkung dieser Thomas-Rotation (naja, wirklich jede beliebige räumliche Rotation) auf einen Spin explizit berechnen. J Objekt für alle J .


[1] https://en.wikipedia.org/wiki/Spin_(Physik)#Higher_spins

[2] https://arxiv.org/pdf/1402.3541.pdf